LSAT For Dummies, 2nd Edition (2014)

Part VI. The Real Deal: Full-Length Practice LSATs

Chapter 16. Practice Exam 1: Answers and Explanations

In this chapter, you find the answers and explanations to the test in Chapter 15 including an example essay for the writing sample, even though it's not scored.

Take your time with these helpful explanations, but also know that you can find an abbreviated key at the end of this chapter if you need to score your test quickly. This chapter also includes information about computing your score.

Section I: Analytical Reasoning

Questions 1–7

As part of your pre-question preparation, set up your game board. Start by assigning your game pieces, designated by the abbreviation of all the guys’ names to single letters (B, C, D, M, and T). If you'd like, create a board that lists potential volleyball games with room for two players each. The conditions don't specify the total number of games, so start with space for five, which you may have to modify as you read through the rules. The rules state that if B plays with M, B can't play with C. Record this rule and its contrapositive: that if B plays with C, B can't play with M. The next rule states that any person who plays with D once must play with M once. Designate “any person” with an X. Remember the contrapositive: that any person who doesn't play with M also doesn't play with D. Record these conditions along with the rules that T plays only once and D and M never play together.

Now, what can you deduce? It looks as though T could play with B, C, or M. But if T can play only one game, he never plays with D because that requires T to play another game with M. You can add this rule to your list.

Lo and behold, you've come up with a decent diagram that looks a little something like this:

9781118678237-un16001.tif

You can continue to consider possible team combinations, such as T can only pair with B, C, or M, but it may be more efficient to save further combination considerations for when you answer each related question.

1. C. Biff and Chas; Chas and Deke; Chas and Marc; Marc and Trip

The first question in the set almost always asks for a complete and accurate list. These are the only questions for which you don't need to refer to a game board. Just look through the options for rule violations and eliminate wrong answers. Focus first on the easiest conditions. Based on the third rule, eliminate answers that have Trip playing more than once. Choice (D) is out. Next, based on the first rule, look for any answers that have Biff playing with both Marc and Chas; that knocks out Choice (A). Now, looking at the fourth rule, does an answer have Deke and Marc playing together? Yes, Choice (B) does, so cross it off. Last, looking at the second rule, does a choice have someone playing with Deke but not with Marc? Yep. Choice (E) has Trip playing with Deke but not with Marc (which, if it happened, would violate rule three). That leaves Choice (C); run through the answer to make sure it works. It has Biff playing with Chas but not with Marc, so that's fine. Trip plays once, with an allowable partner. Chas plays with both Deke and Marc. It all checks out, so Choice (C) is correct.

2. C. five

The multiplication principle tells you that, at most, only ten teams of two can be made up out of five players. The possibilities are BC, BD, BM, BT, CD, CM, CT, DM, DT, and MT.

The first condition states that you can't have both BC and BM, so you're left with eight possible teams.

From the fourth condition, you know DM isn't possible, so there are at most seven possible teams.

The third condition states that T plays on only one team, so you're down to a maximum of five possible teams: (BC or BM), BD, CD, CM, and (BT, CT, DT, or MT). You've deduced that DT isn't possible, which doesn't change the maximum possible team but eliminates DT from the mix. The second condition requires any player who teams up with D also play with M. You can satisfy this condition and maintain a maximum of five teams. To keep BD, you need to add BM by knocking out BC. That gives you at most these possible five combinations: BD, CD, CM, BM, and (BT, CT, or MT).

So the answer is Choice (C).

3. C. Biff and Trip play a game together, and neither Biff nor Trip plays with anyone else.

This question gives you no additional information to work with, so you need to assess each answer choice. Starting with Choice (A), if Biff and Chas play together and no one else, Deke can't play because your game board setup reveals that Deke can't play with Marc or Trip. Choice (A) can't be true. If Biff plays with Marc, then Chas could play with Trip, but that would once again leave no one to team with Deke. According to the second condition, Chas can't play with Deke without also playing with Marc, but the answer choice states that Marc plays with only Biff. Choice (B) is wrong. If Biff and Trip play together, as designated by Choice (C), Chas could play with Deke and Marc. That gives everyone at least one game and violates no rules. So Choice (C) could be true and is likely the correct answer. Check the last two options, though, just to make sure you haven't missed something. If Chas and Marc play together, you once again run up against the Deke rule. Because no one can play with Deke without also playing with Marc, Choice (D) is impossible. If Marc and Trip play together, that causes the same problem — anyone who plays with Deke must also play a game with Marc, so Choice (E) is wrong. Choice (C) is the correct answer.

4. B. Chas and Deke

This question offers another condition, that three players team up with Biff. Refer to your game board and note that B can't team up with both M and C, so the possible three teams created with B can be (BM or BC), BT, and BD. But whoever teams with D must team with M, so B must play with M, T, and D. That eliminates Choice (A). C can't play with B, but he must play. You can't pair him (or anyone else) with T, so Choices (C), (D), and (E) are out. The answer must be Choice (B), which is perfectly acceptable when you consider that the possible team combinations could be BM, BT, BD, CM, and CD.

5. C. Chas and Deke

Consider which player can pair up with all the others. You know it's not Trip; he plays only once. And it can't be Deke because he can't pair with Trip or Marc, which means it also can't be Marc. Biff can't pair with Chas and Marc, so it can’t be Biff. That leaves Chas. When you pair Chas with all four players, you get CB, CD, CM, and CT. Eliminate answers that could be true or must be false. The only pair that must be teamed is Chas and Deke, Choice (C).

6. E. Deke and Trip

Remember that Trip can play only one game, and anyone who plays with Deke is committed to playing a game with Marc, too. So Trip can never play a game with Deke. The answer is Choice (E).

7. D. Marc plays exactly two games.

This question provides an additional rule, that Marc is the only player to team with Biff. Neither Marc nor Trip can pair with Deke, so Chas must play with Deke, and if Chas teams up with Deke, he must also play with Marc. Marc must play with both Chas and Biff, and Chas must play with both Marc and Deke. Either Chas or Marc could play with Trip. The teams that must result from the additional rule appear on the game board:

9781118678237-un16002.tif

Chas and Marc must play more than one game, so Choices (A) and (B) are out. Deke can play only once, so Choice (C) isn't possible. Chas plays no more than three games, which make Choice (E) impossible. The only possible scenario is Choice (D).

Questions 8–12

Before answering the questions, take some time to set up your game board. The game pieces are 2P, 2G, and 2M for ponies, geldings, and mares. Write the six possible horses on the game board. Create six columns, numbered 1 to 6, to mark the positions of the horses as they walk in line. Write in what you know about their placement, marking the positions that must or must not contain particular horses.

Take a moment to consider additional rules. If the third horse can't be a pony, then it must be a gelding or a mare. If the sixth horse can't be a mare, it must be a gelding or a pony. Note these conditions on your chart. Now remember that the wrangler can't put two horses of the same type next to each other. So the first and second horses can't both be geldings, and positions 3 to 6 must contain one gelding. This rule affects all your computations; keep track of the restrictions on your diagram.

9781118678237-un16003.tif

That's your permanent diagram. Be sure you give yourself a little room to jot down possibilities. You may be able to come up with a few other conditions, but moving on to the questions as quickly as you can is important. Before you do, note that having a gelding in the second position provides more restrictions than a gelding in the first position.

8. E. The fifth horse is a pony.

The question offers no additional rule, so you need to consider the possibilities set forth by each answer choice. The answer cannot be true, so eliminate choices that could or must be true. Write the possible orderings you come up with next to each answer choice; you may refer to them again. Try Choice (A). If the first horse is a pony, then the second one must be a gelding, and the third must be a mare; if the sixth horse is a gelding, then the fourth can be a pony and the fifth a mare: PGMPMG. That works. Now Choice (B). You can do that one: MGMPGP. You know Choice (C) works because the second horse is a gelding in the ordering you came up with in Choices (A) and (B). The same holds for Choice (D). The orderings you came up with for Choices (A) and (B) place a mare in the third position. That leaves Choice (E). It's likely correct, but try it out anyway just in case you've missed something. If the fifth horse is a pony, the last one must be a gelding because it can't be a mare. The fourth horse must be a mare because it's next to a pony, and you need your other gelding for one of the first two slots. Then you run into a problem: The third horse can't be a pony. It can't be a mare because that puts a mare next to another mare, and it can't be a gelding because the other gelding has to go in one of the first two spots. The game board with the lineups for each answer looks like this:

9781118678237-un16004.tif

Putting the pony in the fifth spot results in an impossible lineup, so Choice (E) is your answer.

9. D. The fifth horse is a mare.

Consider the new rule this question provides. If the third and fifth horses are the same type, what can't they be? They can't be ponies because the third horse can't be a pony. They can't be geldings because you have only one gelding to use in the last four spots. So they must be mares. Take a look at the answers. The correct choice for this “must be true” question has to be Choice (D). The other answers all could be true, but they don't have to be; work them out if you want.

10. A. The first horse is a gelding.

This question is easy, or it should be if your diagram looks right and you understand the rules. If the third horse is a gelding, then what happens to the lineup? You know the first must be a gelding because you can't place a gelding in the second position next to the other gelding, and you have to have a gelding in position 1 or 2. The correct answer has to be Choice (A). You can test the other answers if you want, but Choice (A) is the only one that must be true.

11. E. the third and last horses

You could plug the same horse into all those pairs and see what you get, but there's a faster way to work this one. Look at your diagram. You know the third horse can't be a pony, and you know the sixth horse can't be a mare. You also know they both can't be geldings because you need one gelding to occupy the first or second position. So the third and sixth horses can't be the same type. That's Choice (E).

If you want to test the possibilities, try this: Make both members of the pair designations in each answer choice the same kind of horse and see whether that works out. First Choice (A): You know from your answer to Question 10 that both the first and third horses can be geldings, so that's wrong. Try Choice (B). If you make the first and fourth horses geldings, you could have this perfectly legal lineup: GPMGMP. Now take a look at Choice (C). If you make both the second and fourth horses ponies, you get GPMPMG. That's fine. (You can also make them both geldings, but that would be overkill.) Try Choice (D). You could have MPGMPG, which is fine. Choice (E) has to be the right answer.

12. D. fourth, sixth

Don't you hate it when the rules change? Just think of the poor wrangler! Anyway, here's how to work this one. To accommodate three ponies and still keep each separated from the other (pony rivalry?), you have to put one in the middle, in position 3 or 4, but you can't put a pony third, so you have to put a pony fourth. Cross out Choice (A) because it doesn't list the fourth position. The two remaining ponies have to be at least one position away from fourth. Only one position to the right of fourth fulfills that requirement: the sixth. Eliminate Choices (B) and (C) because they don't include the sixth position. From the remaining Choices (D) and (E), you know that you need to evaluate just the second position. The third pony can be first or second as long as a gelding takes the other spot.

9781118678237-un16005.tif

So the spots that must be occupied by ponies are the fourth and the sixth positions, Choice (D).

Questions 13–19

Creating the right diagram takes you far with seemingly complicated grouping problems like this one. In this case, you have seven players to place in one of three locations. Start with what you know. First, list the game pieces: B, C, M, R, S, Y, and Z. Your places are f, l, and o. (Tip: Make the places lowercase so you don't mix them up with people.) Then set up a quick chart of the three tournament locations and shortcuts for each of the rules.

9781118678237-un16006.tif

You can start making some conclusions at this point. If twice as many people go to Fargo as go to Omaha, then there are two possibilities: Either one person goes to Omaha and two to Fargo, leaving four to go to Little Rock; or two people go to Omaha, four go to Fargo, and one goes to Little Rock. That's useful. Write that down in a little template, adding the useful permanent information about C always going to Fargo and Y never going to Fargo:

9781118678237-un16007.tif

After you have this framework in place, plugging in possibilities is simple. You can make a few deductions just by looking at your game board. For example, if B and R must attend the same tournament, then their possible locations are quite limited. They can fit only into locations with at least two open slots, and even then you have to allow space for the people that have to be separated. So realistically, B and R can either attend the conference in Little Rock when it has four slots (it's the only one with two open spots in that first possibility) or the conference in Fargo when it has four slots. That's good to know. What about the two slots allocated to Omaha in the second scenario? B and R can't have them; you have two pairs of members that have to be split up — S and Y, and M and Z — and Little Rock has only one slot open, so you need an Omaha slot for one of those pairs. Update the game board with this info:

Now that is a diagram! And instead of an unlimited number of scenarios, you now have two discrete possibilities, each of which is nearly half complete.

13. B. Maev attends the tournament in Fargo.

Consider the new temporary rule provided by this question. How can Brunhild and Yorick attend the same tournament? According to your game board, they can attend together only if they both go to the tournament in Little Rock in the first option. That leaves one slot open in each of the cities to be occupied by Maev, Solomon, and Zilla. Now go through the answer choices and eliminate the one that must be false. Choice (A) is wrong because Brunhild must be in Little Rock. Choice (B) could be true; Maev could take the open slot in Fargo, leaving space for Zilla in Little Rock:

9781118678237-un16008.tif

Choice (C) is wrong because Roxanne is with Brunhild in Little Rock. Choice (D) is wrong because Solomon can't attend the same tournament as Yorick, and Yorick must be in Little Rock. Choice (E) is wrong because Yorick must be in Little Rock. That leaves Choice (B) as the only possible answer.

14. B. Solomon is the only team member who attends the tournament in Omaha.

You can eliminate several answers just by looking at your original game board. Choice (A) can't be true because Roxanne can't attend a tournament without Brunhild. Choice (C) is wrong because either two or four people must go to Fargo. Choices (D) and (E) are wrong because there's no way that either two people can go to Little Rock or three to Fargo. So could Choice (B) work? Yes, it's possible for the team to send only one member to Omaha, and there's no reason it can't be S. Choice (B) is correct.

15. A. Brunhild attends the tournament in Fargo.

This question tells you to use the second option on your original game board. When you know which option to use, you don't have to do much to figure out the answer. If one member goes to Little Rock, then four go to Fargo and two to Omaha. Who has to go to Fargo in that scenario? Cayenne, Brunhild, and Roxanne. Choice (A) looks like your answer. Skim the other choices anyway — they're all possible, but none of them is required by the rules. Choice (A) is correct.

16. A. Brunhild and Roxanne

Rephrase this exception question so that you remember to choose the answer with the members who can't go to Omaha. Choice (A) includes Brunhild, whose only options are Little Rock and Fargo and thus can never sample the delights of Nebraska's largest city. Look over the other choices to make sure no one else is prohibited from attending the Omaha tournament. They all look fine, and they're all pairs, which is certainly possible. (If any choice had three members, that would have been a dead giveaway right answer.) So Choice (A) is correct.

17. C. Brunhild, Solomon, and Zilla: the tournament in Little Rock

Eliminate Choices (A) or (B); Brunhild can't go to Omaha. Your game board's first option indicates that you could add Solomon and Zilla to Little Rock and maintain all conditions. Choice (C) doesn't obviously violate anything yet, so skip it for the moment to see whether Choices (D) and (E) have clear problems. Choice (D) does; Yorick can't go to Fargo. Choice (E) doesn't work either. When you refer to the second option on your game board, you see that space for only one more participate remains. Either Zilla or Solomon can participate in Fargo but not both. That leaves Choice (C) as the only legal possibility. Test it out completely just to be sure. If you put Brunhild, Roxanne, Solomon, and Zilla in Little Rock, then Yorick can go to Omaha, and Maev can go to Fargo, which separates the incompatible people and keeps Yorick away from Fargo.

9781118678237-un16009.tif

That works, so Choice (C) is right.

18. B. Cayenne, Yorick, and Zilla do not attend the same tournament.

Eliminate answers that could be true or must be false. Choice (A) is wrong because it could be true. In the second option, Brunhild and Cayenne can attend the same tournament in Fargo. Choice (B) must be true. Yorick and Cayenne can't be together because Cayenne must go to Fargo and Yorick never goes to Fargo. Check the other answers just to be sure. All are possible, but none of them must be true. You can put Maev and her various partners together, but you can also separate them. Choice (B) is correct.

19. D. Maev, Solomon, Yorick

Your game board indicates that either two people go to Fargo and five go elsewhere or four go to Fargo and three go elsewhere. That means any answer that doesn't have either three or five choices is wrong; eliminate Choices (A) and (B) immediately. Choices (C), (D), and (E) have three people listed, which means you're looking at the second option on your game board. In that option Cayenne, Brunhill, and Roxanne must go to Fargo. Choices (C) and (E) are both wrong because they both include Roxanne. That leaves Choice (D). Could Choice (D) work? Sure. Send Zilla to Fargo, send Solomon to Little Rock, and send Yorick and Maev to Omaha and you've observed all rules:

9781118678237-un16010.tif

Choice (D) is correct.

Questions 20–26

This kind of open grouping problem with ordering can be tricky. However, this problem is easier than you may think when you keep the rules in mind.

remember.eps Start with what you know. List the seven game pieces: C, C, C, V, V, V, and V. You have five cups in a row, and each cup contains one or two scoops. Create a game board with columns 1 through 5 and two spaces under each column. You don't really know where the row begins or ends; that means you can number just as well from right to left as from left to right. It's totally arbitrary. Add your topping possibilities: s, s, n, n, n, n, and n.

The rules don't give you any permanent homes for these ice creams or toppings. Either flavor of ice cream can get either topping. You must remember to account for every flavor and every topping. Every possible scenario must include four vanillas, three chocolates, two sprinkles, and five nuts. When you plug in data, you can account for both flavor and topping using a system of uppercase and lowercase (for example, vanilla with sprinkles would be Vs, and chocolate with nuts would be Cn). Write the conditions in shorthand underneath your chart so your game board looks something like this:

9781118678237-un16011.tif

Before you jump into the questions, think a little bit. You know every cup can hold up to two scoops. With only five cups and seven scoops, at least two cups will have two scoops. Does every cup need a scoop of ice cream in it? Not necessarily. You don't know how many teenagers are sharing this frozen monstrosity; maybe there are only four of them, and each one gets his or her choice in an individual cup, leaving one cup empty. Here's another thought: You can't mix flavors in one cup, but you can mix toppings. So you can put two scoops of vanilla in one cup, and top one with nuts and one with sprinkles.

20. D. 1: nothing; 2: two scoops of vanilla with nuts; 3: two scoops of chocolate with nuts; 4: two scoops of vanilla with sprinkles; 5: one scoop of chocolate with nuts

For questions that ask for the complete and accurate arrangement, you simply need to look for obvious rule violations. Check answers for ones that list more than two scoops per cup. Choice (A) has three scoops in cup 5, so it's no good. Look for answers that don't give you three chocolate and four vanilla. Choice (E) has four scoops of chocolate, which is too many. Consider the rule that there are two sprinkle toppings and five nuts. Choice (C) has three scoops with sprinkles, so it's wrong. The first condition specifies that no nut-topped scoop can sit next to a sprinkle-topped scoop. Choice (B) has a vanilla with nuts in cup 2 next to a chocolate with sprinkles in cup 3; that doesn't work. Choice (D) must be the answer. It maintains all conditions: you have four vanillas, three chocolates, five nuts, and two sprinkles, and you don't have either vanilla and chocolate intermingled in the same cup or vanilla with nuts next to chocolate with sprinkles. That looks good. Choice (D) is the answer.

21. A. At least one cup contains exactly one scoop of chocolate ice cream.

First, note that you're looking for something that can't be false, which is another way of saying that it must be true. Now consider the answers. Choice (A) looks like a good option right off the bat because it contains the fewest restrictions. If you have seven scoops of ice cream, and the most scoops you can put in the same cup is two, and three of those scoops are chocolate, and you can't put vanilla and chocolate in the same cup, then inevitably, at least one cup will contain nothing but one scoop of chocolate ice cream and its topping.

9781118678237-un16012.tif

Choice (A) is obviously the correct answer, and you can save time by spotting it and moving on. Just for the sake of study, consider the other answers. Each of them is in fact possible but doesn't necessarily have to be the case. This ice cream sundae is pretty flexible. Remember, an answer that could be true but isn't necessarily true isn't good enough; it must be true to be correct. Choice (A) is correct.

22. C. three

You already know the answer can't be Choices (D) or (E). You run out of cups before distributing all seven scoops. Choice (C), three, looks like a good answer; test it to see whether it's possible. If you put two scoops of vanilla with nuts in cups 1 and 2, then a scoop of chocolate with nuts in cup 3, followed by single scoops of chocolate with sprinkles in cups 4 and 5, you have three cups each containing exactly one scoop of ice cream. You don't need to bother with Choices (A) and (B). Choice (C) is correct.

23. E. All the scoops of vanilla are in the second and fourth cups.

This question offers another temporary condition. If the third cup is empty, that leaves you only four cups to hold all seven scoops of ice cream. Because you have four scoops of vanilla, you'll have to have two cups that each contain two scoops of vanilla, which leaves one cup with one scoop of chocolate and one cup with two. To separate the two cups of vanilla, you have to put one on either side of cup 3 (either cup 1 or cup 2 will hold 2 scoops of vanilla, and cup 4 or cup 5 will hold the other two). The chocolate scoops occupy the cups not occupied by vanilla. You can't put sprinkles on a scoop of chocolate; you can't avoid putting chocolate and vanilla in adjacent cups, and some of the vanilla must have nuts on it. Here are a couple ways you could arrange the sundae:

9781118678237-un16013.tif

Now look at the answers. Eliminate choices that must be true or must be false. Choice (A) must be true; one cup contains one scoop of chocolate. Choice (B) must be true; you can't arrange this sundae without putting a scoop of chocolate immediately next to a scoop of vanilla. Choice (C) must be true; you can't put sprinkles on any of the chocolate scoops because they all have to be in cups next to cups with scoops of vanilla. Choice (D) must be true. Choice (E) is the only answer that doesn't have to be true. The vanilla scoops could be in the second and fourth cups, but they could also be in the first and fifth, or the first and fourth, or the second and fifth. Choice (E) correct.

24. D. A scoop of chocolate with nuts is in the fifth cup.

Add the new temporary condition to your game board.

Notice that with two sprinkles accounted for, all the other scoops must have nuts on them. That means you still have to place a Cn, Vn, Vn, and Vn.

9781118678237-un16014.tif

Apply the possibilities to your game board and eliminate answers that violate the conditions. Try Choice (A). If you put chocolate with nuts in cup 1, then you have a problem because you have to put a vanilla with nuts in either cup 3 or cup 5, if not both, but both of them are adjacent to the chocolate with sprinkles in cup 4, so Choice (A) doesn't work. Choice (B) has to be wrong because vanilla is already in the second cup and you can't mix the flavors in the same cup. Choice (C) is also wrong because that puts three scoops in one cup. Now try Choice (D); could you put a scoop of chocolate with nuts in cup 5? Sure, and then you could fit two scoops of vanilla with nuts in both cups 1 and 2 and satisfy all the rules. Choice (D) looks like your answer, but check Choice (E) anyway. Could you put two scoops of ice cream in cup 5, like Choice (E) suggests? No. The two scoops would have to be vanilla because you only have one scoop of chocolate left. All the remaining vanilla scoops have nuts on them, and you can't put Vn next to Cs. Choice (E) must be false, and Choice (D) is correct.

25. C. the third cup

If each scoop of chocolate gets its own cup, then the four scoops of vanilla must occupy only two cups. The scoops of vanilla have nuts on them, which means two of the scoops of chocolate have sprinkles on them and therefore can't be next to vanilla. So, in no particular order, your cups will be: VnVn, VnVn, Cn, Cs, and Cs. To satisfy the first rule, you have only one way to ensure that vanilla with nuts doesn't get put next to chocolate with sprinkles: Separate them with the one remaining scoop of chocolate, which must have nuts on it. That scoop of chocolate with nuts has to go in the center cup, cup 3. Then you could put the vanilla in cups 1 and 2 and the chocolate with sprinkles in cups 4 and 5; or the vanilla could go in cups 4 and 5 and the chocolate with sprinkles could go in cups 1 and 2. Cup 3 is the only place vanilla can't go, so the answer is Choice (C). Here's the diagram:

9781118678237-un16015.tif

26. E. 1: one scoop of vanilla with sprinkles; 2: empty

Test the answer choice possibilities on your game board and eliminate those that are possible. All you really have to do is sketch in the contents of the first two cups and then mentally see whether you can fill in the rest; you don't need to plot out the whole thing if you can see how to work it out easily. (Yes, we know we ordinarily say to diagram everything; in this case, though, running through the possibilities in your head is quicker, at least for most of the answers. If you get confused, though, by all means keep writing everything down. All we're suggesting is that if you can work part of a problem mentally without getting confused and without hurting your accuracy, doing so is okay and saves you time.) Try Choice (A). Choice (A) works fine. If you put the sprinkles on two of the remaining three scoops of vanilla, distributing the rest of the ice cream would be easy. Consider Choice (B), which puts all sprinkles on top of the vanilla scoops, so you don't run into pesky rules about neighboring scoops. Cross out Choice (B) and check Choice (C). You may want to draw this one out on your game board because the answer allocates only one scoop of ice cream. This scenario works fine if you allocate the scoops like this: 1: empty; 2: Cn; 3: VsVs; 4: CnCn; 5: VnVn.

9781118678237-un16016.tif

Now try Choice (D): Sure, Choice (D) works. You don't have any chance of a chocolate with sprinkles being put next to a vanilla with nuts, so you can just distribute the rest of the scoops as you see fit. That leaves Choice (E), which looks like this:

9781118678237-un16017.tif

Uh-oh. You run into a problem here. The problem isn't the sprinkles; you could put them on another scoop of vanilla. The problem is that you have three scoops of vanilla, three scoops of chocolate, and only three cups to put them in. You can't place them without violating the condition against mixing flavors in one cup. Choice (E) is impossible and therefore is your answer.

Section II: Logical Reasoning

1. D. Christie's states in its auction catalogs that all buyers should consult outside specialists before bidding on antiques, especially in the case of extremely valuable items.

You need to weaken the argument. Tempest claims that, based on expert opinion, her urns aren't 18th century, and therefore, Christie's advertised them falsely. To weaken this conclusion, look for evidence that would suggest Christie's catalog copy was not false advertising. Choice (A) actually strengthens Tempest's argument by bolstering her evidence that Christie's advertised the urns as 18th century. Choice (B) doesn't directly address the issue of whether Christie's advertised the urns falsely. The general difficulty in dating antiques doesn't help Tempest's case, but it also doesn't help exonerate Christie's in this particular case. Tempest's argument doesn't refer to scientists, so Choice (C) is unlikely to strengthen or weaken Tempest's argument, which is after all about Christie's truthfulness, not methods of dating urns. Choice (D) is a very good answer because it specifically addresses the Christie's advertisement. If Christie's told Tempest upfront that she shouldn't rely exclusively on the claims in its catalog, then she can't blame the auction house if she didn't have her urns assessed before purchasing them. Choice (E) isn't relevant because it regards the integrity of the experts but not the integrity of Christie's advertisement. Choice (D) is correct.

2. C. Recent studies have shown that a large proportion of criminals sentenced to death are later shown to be innocent of their crimes.

This is a “weaken” question. The argument's conclusion is that those convicted of heinous crimes by juries deserve the death penalty because such criminals are incorrigible and the families of victims need them to die for closure. Note, though, that the conclusion says “anyone whom a jury convicts,” which isn't necessarily the same group of people that the evidence talks about — that is, those “who commit coldblooded, premeditated murders.” What if someone is convicted wrongly? Choice (A) doesn't weaken the argument. The author uses family feelings as evidence to support the conclusion. Choice (B) is an advantage of using the death penalty and doesn't hurt the argument. Cost, by the way, doesn't enter into the argument at all. Choice (C) does weaken it. If large numbers of convicts are in fact innocent, then they're not incorrigible extreme criminals, and they don't deserve to be executed. Choice (D) is evidence of how pleasant the death penalty can be, and it doesn't hurt the argument. Choice (E) is not relevant because the author's argument isn't based on the actions of other states. Because the author concludes that the death penalty should continue because convicted murders deserve it, Choice (C) is the best answer.

3. D. assumes that the only reason anyone would attend law school is to acquire a job at a top-ranking, high-paying law firm

The problem with this argument is that it suggests that there's no point in attending any law school that's not in the top ten, because those are the only schools that lead to jobs in high-paying firms. But working for a top-ranking, high-paying law firm may not be the only reason to go to law school. Choice (A) is wrong because the argument doesn't promise anything. Choice (B) doesn't work because the argument doesn't address quality of education. Choice (C) is wrong because nothing in the argument implies that the author thinks top-firm recruiting practices are good or bad — there aren't any value judgments here, so you can't attack them. Choice (D) is your answer. Surely, some people can think of reasons to attend law school that go beyond finding a job in a top-ranking law firm. Choice (E) isn't relevant. You can't identify any misrepresentation based on the information contained in this argument. Choice (D) is correct.

4. C. Birds are an indicator species of West Nile virus; the virus always appears in birds before humans start catching the disease.

The exterminator's conclusion is that there probably won't be an outbreak of West Nile virus this summer based on the fact that no one has found dead birds carrying the virus. That would only be good evidence if the virus almost always appears in birds before attacking humans, so that's the assumption you're looking for. Choice (A) isn't relevant because whether birds are more susceptible to the virus doesn't tell you anything about whether they're a predictor of human infection. Choice (B) doesn't establish a connection between the virus, birds, and humans. Choice (C) is a good answer. If birds are an indicator species, and the virus hasn't killed any birds yet, that would be good evidence that an outbreak among humans is unlikely. Choice (D) goes against what the exterminator assumes because it suggests that a human outbreak is actually possible, even though the birds show no evidence of the virus. Choice (E) is completely irrelevant. The effect of insecticide on birds has no bearing on whether a bird outbreak predicts a human outbreak. Choice (C) is the correct answer.

5. A. The editorialist believes that stem cell research offers many powerful benefits and that the arguments of its detractors are invalid because they misunderstand the nature of stem cells.

The editorialist argues that stem cell research could be beneficial and that it's not akin to abortion because stem cells aren't embryos. Choice (A) summarizes her statements nicely. The remaining choices are off base. Choice (B) is the opposite of what the editorialist thinks; she claims that stem cells aren't and can never be embryos. Choice (C) may be something the editorialist would agree with, but nothing in her statements suggests she believes that religious groups have no business influencing legislation. Choice (D) is wrong because the editorialist never suggests banning research on a particular type of stem cells. Choice (E) is also wrong; the editorialist believes the benefits of the research are such that the research should be legal. Choice (A) is correct.

6. C. DNA samples taken from the heart in question and from hair cut from Marie Antoinette's head when she was a child indicate that the heart and hair sample came from people who were closely related.

Some scientific evidence — DNA or something like that — proving conclusively that the heart was Louis XVII's would support the argument. Choice (A) doesn't necessarily help because what was “traditional” doesn't necessarily apply to every situation; an impostor's heart could have been removed and preserved to fool Louis's enemies in this exceptional case. Choice (B) is scientific evidence that tells you the age and sex of the heart's owner, but that information doesn't rule out an 8- to 12-year-old male commoner. Choice (C) supports the argument. If Louis's mother and the heart have similar DNA, that would be scientific proof of the heart's identity. Choice (D) is an interesting factoid but doesn't establish that the heart belonged to Louis. Choice (E) could be evidence that the heart belonged to a child who died in prison but doesn't help identify who that child was. Choice (C) is the best support for the conclusion.

7. D. early specialization in a single sport is beneficial to children

Emilio and Julienne disagree about whether early specialization in youth sports is beneficial or harmful to children. Only Julienne mentions teamwork, leadership, and enjoyment of sports, which eliminates Choices (A) and (B) as bones of contention. They both seem to think that kids should play sports, so Choice (C) is out, and they don't discuss an age below which kids should play sports at all, so Choice (E) is wrong. Choice (D) is the answer; Emilio thinks early specialization is good, and Julienne thinks it's bad.

8. A. Most home freezers can store food for no more than three months.

Okay, if food at 0 degrees lasts a year, food at 5 degrees should last six months, and food at 10 degrees should last three months. That looks like Choice (A). Choice (B) doesn't necessarily follow from the statements because they say nothing about homeowners’ knowledge. The statements don't address safety, only flavor, so Choice (C) doesn't have to be true. Choice (D) isn't a necessary implication of the statements because they say nothing about commercial freezers. Likewise, the statements provide no information about how long homeowners actually store their frozen food, so Choice (E) isn't relevant. Choice (A) is best.

9. E. A botanist finds a new kind of cactus in the driest part of the desert. All plants that live in the dry desert are extremely efficient at collecting and retaining water. Therefore, this new cactus must be extremely efficient at collecting and retaining water.

The pattern goes like this: mollusk = deep; all deep = predator; therefore mollusk = predator. You can rewrite that as A = B; all B = C; therefore A = C. Choice (A) goes like this: enthusiast (A) finds studio (B); studio (B) = experienced instructors (C); therefore enthusiast (A) finds good instructors (D). “Experienced and highly trained” isn't necessarily the same as “good,” so Choice (A) isn't quite right. Choice (B) goes as follows: player (A) = Croatia (B); most Croatian players (B) = tall and hardworking (C); player (A) = tall and hardworking (C). The original argument qualifies the second premise as “all” B = C, but the answer choice qualifiers the second premise as “most” B = C. This is not a match. Choice (C) goes like this: tomato (A) = garden (B); all tomato (A) = nightshade (C); therefore tomato (A) = nightshade (C). That's not much of a conclusion. To match the given argument, the second statement would have to say something about all things found in the garden. Choice (D) reasons as follows: A = perfect sapphire; all sapphire (A) = hard and durable (B); therefore perfect sapphire (A) = hard and durable (B). This answer doesn't provide you with a “C” element in the argument. Check out the reasoning in Choice (E): cactus (A) = desert (B); all desert (B) = efficient with water (C); therefore cactus (A) = efficient with water (C). That duplicates the pattern stated in the argument, so Choice (E) is correct.

10. B. A wife wants her husband to wash the dishes, take out the garbage, and make the bed in the morning, but she knows he won't do all three. She most dislikes taking out the garbage herself, so she makes a point of asking him to do that many times and ignores the dishes and bed.

Look for an answer that involves someone who chooses one battle that matters the most to him out of several possible less important ones. Choice (A) is no good because the sister isn't prioritizing one battle out of many; the scenario addresses no battle other than the gas tank scuffle. Choice (B) looks like a good choice. The wife chooses to ask her husband to complete the one task that she cares most about out of several tasks she'd like to see done. Choice (C) illustrates the dangers of trying to fight two battles at once, so it's wrong. Choice (D) is an example of someone not fighting a battle at all but running away instead. Choice (E) doesn't fit because it doesn't present a choice of battles, only different paths to the same goal. Choice (B) is the best answer.

11. D. The cheap foods that make up the majority of the diets of poor people are full of calories but devoid of essential nutrients.

Look for an answer that shows why people who spend little on food still get fat. Make sure there's a connection between money and food. Choice (A) explains nothing about the surprising paradox that some people who live in poverty experience both malnutrition and obesity. Choice (B) is a possibility but doesn't really explain why obesity is the result of this lack of nutritional knowledge instead of, say, starvation. Choice (C) doesn't focus on impoverished people, so it doesn't explain why they in particular are malnourished and obese. Choice (D) does resolve the paradox. If affordable food is full of calories, it makes people fat, and if it's devoid of nutrients, it can result in malnutrition. Choice (E) doesn't address poor people specifically, so like Choices (A) and (C), it doesn't resolve the paradox surrounding their particular obesity and malnutrition. Choice (D) is the answer.

12. C. Most people use Windows computers. Therefore, software producers should not bother to make any software for Macintosh computers.

The reasoning goes like this: most players = male; manufacturers shouldn't bother with females (non-males). Choice (A) reasons that most cyclists = male, so manufacturers should concentrate on males. That's close but not right because it states what manufacturers should do with males rather than what they should not do with females. Choice (B) states that most Americans = cellphones, so most Americans shouldn't bother with landlines. This argument gets that something should not be done, but it states what consumers shouldn't do rather than what manufacturers shouldn't do. Choice (C) reasons that most users = Windows, so software producers shouldn't bother with Macintosh. That follows the original line of reasoning. Choice (D) goes most lawyers = glasses, so optometrists should target lawyers. The original argument concludes with what an industry should not do for its less common customers rather than with what it should do. Choice (E) states that Labs = easy to train, so a person who wants an obedient dog should consider Labs. There's no parallel at all here to a situation about manufacturers and their various customers. Choice (C) is correct.

13. D. Because drivers talking on cellphones are distracted, they are more prone to get into accidents.

You're looking for an assumption. Assumptions connect the premises to the conclusion. The conclusion is that if drivers can't talk on cellphones, the roads will be safer, based on the evidence that drivers on phones are distracted. That means the author assumes that distracted drivers are unsafe drivers. (Yes, that's obvious, but the author doesn't explicitly say what the connection is, so it's an assumption.) Choice (A) is tricky and could be a possible answer if you were asked to support the legislator's argument, but information about the results of a specific cellphone use study is way too specific to be the legislator's assumption. Assumptions are rarely based on particular statistical data. See whether you can find a better answer. Choice (B) is wrong; the author probably does assume that this is the legislature's job, but this assumption isn't necessary to the specific argument connecting distraction and road safety. Choice (C) doesn't work; the author isn't concerned with distinctions among types of cellphones users. Choice (D) is something the author assumes — it explains the connection between cellphones and distracted drivers and dangerous roads. Choice (E) contains mostly irrelevant details about other behaviors, and to the extent it addresses drivers on cellphones, it just reiterates evidence the argument already explicitly states. Choice (D) is the right answer.

14. A. equates time spent on a job with quality, without examining the products to verify the quality

The partner draws a direct correlation between time spent on a task and quality of work, which isn't in fact a foregone conclusion. Choice (A) looks like a good answer. The partner's reasoning is flawed because he assumes the work that took longer is better without even reading the briefs to compare them. Choice (B) isn't the flaw. He's not criticizing Attorney B and, if anything, is happy with her performance. Choice (C) doesn't work because the partner does explain why he thinks Attorney B's brief is better. Choice (D) doesn't quite work because the argument never recommends keeping track of time as a general practice. Choice (E) is irrelevant. Therefore, Choice (A) is correct.

15. E. assumes incorrectly that a characteristic found in the majority of a group must also be found in the majority of each subgroup within that group

The student faultily assumes that because most insects have wings, most ants also must have wings to qualify as insects. But in fact, there's nothing inconsistent about most insects having wings and very few (or even no) ants having wings. There are so many other winged insect types out there to make up the difference and maintain the truth of the teacher's definition (“in most cases wings”). Choice (A) is wrong because the student actually does consider this possibility. Choice (B) is wrong because the student's conclusion takes into account his reliance on his teacher's accuracy. Choice (C) is an irrelevant criticism. The student's argument is merely that ants aren't insects; making this argument doesn't require him to say what they are instead. Choice (D) is both inaccurate and irrelevant. The student says nothing about the significance of wings, and it wouldn't matter anyway. You're only interested in the presence or absence of wings, not in their uses. Choice (E), on the other hand, points out the student's reasoning error clearly. It's just a bit tricky because it states the problem in general terms, without referring specifically to ants or insects, but that doesn't make it any less correct as a statement of the student's flaw. Choice (E) is the best answer.

16. C. It fails to realize that spending money on discounted items is still spending money and will result in no savings at all.

Ah, the more you spend the more you save, or maybe the more you spend the more you spend. Spending money, even at a discount, is spending, not saving. Choice (A) is wrong. All the merchandise is 25 percent off, so the speaker isn't wrong about that. Choice (B) is wrong because the author doesn't make any claims about the actual dollar prices of anything. Choice (C) is an area in which he is vulnerable to criticism; he doesn't save anything if he spends money. (He may get better prices on items he would have bought anyway, thereby paying less than he might have, but he's still spending, not saving.) Choices (D) and (E) don't work because the reasons for the sale aren't relevant. Choice (C) is correct.

17. B. The high deductibles of consumer-driven health insurance plans will lead to higher healthcare costs.

Christina thinks that the high deductibles of consumer-driven health plans save money by minimizing the use of doctors, and Guthrie disagrees, arguing that they may actually cost more in the long run because untreated conditions could get much worse. Christina doesn't address the topic of Choice (A), so it can't be a point of disagreement. Choice (B) looks like a good possibility; they do disagree on whether raising deductibles will lower (Christina) or raise (Guthrie) costs in the long run. Choice (C) can't work because neither discusses the relationship of business size to health insurance affordability. Choice (D) presents an area where the two agree rather than disagree; they both think high deductibles will keep people away from the doctor. Choice (E) refers to a minor aspect of only Christina's argument, on which Guthrie has nothing to say, so it can't be their area of disagreement. Choice (B) is correct.

18. B. Even well-known and widely available information, provided in an unexpected context, can be misleading.

The argument concludes that meaning of language often derives from context and that people may not recognize simple concepts when they encounter them in a new or unusual situation. Choice (A), though possibly true, isn't the conclusion. The Internet discussion board is offered only as an example of the importance of context. Choice (B) mirrors the summary of the conclusion, so it's likely the correct answer. Choice (C) is too general. You know that because of the inclusion of the word “always.” Additionally, the speaker's primary objective is to comment on a phenomenon rather than give advice on how to deal with it. Choice (D) is wrong because the participants obviously do know about the popular movie. More to the point, the main conclusion is about context in general, not about movies, perfumes, or the Internet, which are only part of an example in support of that conclusion. The same for choice (E); perfume names are only part of the author's example. Choice (B) must be correct.

19. A. The higher concentration of oxygen in nitrox becomes toxic when it is compressed at increasing depths.

On the one hand, nitrox is safer than air. On the other, it's more dangerous than air. What's the difference? Note the last sentence, in which the divemaster describes nitrox's hazards. He says nitrox is dangerous “at greater depths that are safe for divers using air.” So the apparent discrepancy must be due to nitrox's behavior at depths, and something about going deep makes it dangerous. Choice (A) looks like a good explanation for this phenomenon. If oxygen gets dangerous when compressed, that explains why nitrox is hazardous at depths that don't make air especially dangerous. The divemaster never mentions anything about different behaviors exhibited by nitrox divers, so you can't assume Choices (B) or (C) offer an explanation for the apparent paradox. Choice (D) sounds like a real hazard, but it's not connected with the seeming discrepancy surrounding nitrox's safety because all divers face this bacteria risk. Choice (E) would, if anything, make nitrox sound even safer and doesn't begin to explain why it's more dangerous than air in some circumstances. Choice (A) is correct.

20. E. It is a statement of fact about which the author goes on to make an opinion.

The claim in the question is a statement of fact in the first sentence that the speaker explores more thoroughly, concluding with the statement that it's a mistake. She's not suggesting that it's a good thing, so Choices (A) and (D) are wrong. It's not evidence, so Choice (B) is wrong. Her actual conclusion is that not taking vacation is a mistake, so Choice (C) is wrong. It's a statement of fact that provides the background information for the author's opinion about what Americans should do with their vacation time, so Choice (E) is correct.

21. D. A class taught by a PhD, even in a lecture format with hundreds of students, is a better learning environment than a smaller class taught by an instructor without a PhD.

The author concludes that having classes taught by PhDs instead of instructors improves students’ learning experience, so he must assume that PhDs, even in very large classes, are somehow better at teaching than instructors in smaller ones. Choice (A) isn't his assumption; in fact, the reference to increased class sizes suggests that the university won't be hiring more faculty. Choice (B) contradicts the author's conclusion, assuming that student participation is a good thing. Choice (C) is interesting information but isn't essential to the conclusion about the practices of the particular Southern state university, especially because it doesn't mention students’ learning experiences. Choice (D) looks like the right answer. The author does seem to think that a class taught by a PhD is somehow superior, which explains why he thinks the change would benefit students. Choice (E) explains why the university may want to increase classes taught by PhDs, but it doesn't explain how that would benefit students. Choice (D) is correct.

22. E. Griselda and Theodore disagree about whether the United States should respect the opinions of other nations when making decisions that affect national security.

Griselda cares what other countries think of the United States; Theodore doesn't. Griselda doesn't say anything that would give the impression that she thinks the United States sometimes should send out troops despite international disapproval, so Choice (A) is wrong. Choice (B) overstates Theodore's position; he doesn't reject war when in the national interest, but he also never says it's always the proper response. Choice (C) is wrong. Theodore doesn't think the international community's opinion is important. Choice (D) is also wrong. Griselda doesn't mention the United Nations, so you can't assume that she thinks this. Choice (E) is correct because it sums up the conversation nicely.

23. A. assumes that consuming any amount of alcohol during pregnancy will cause birth defects and mental retardation

The obstetrician draws a connection between all alcohol consumption and pregnancy that may not be entirely justified by the facts. Heavy drinking may always be bad, but he provides no additional information to show that light drinking is equally bad or will definitely cause birth defects. Choice (A) looks like a good answer. The obstetrician is assuming that any amount of alcohol is equally deleterious, when in fact his evidence has only connected birth defects with heavy drinking. Choice (B) isn't as good. The doctor doesn't mention prevention or suggest that women be forcibly stopped from drinking, merely that they should choose not to drink during pregnancy. Choice (C) doesn't work because the doctor doesn't mention the state or its interest in children's health. Choice (D) isn't really a flaw. The argument doesn't suffer from lack of proof; it is the unwarrantedly broad conclusion the obstetrician draws from the evidence he does provide that's the problem. Choice (E) would be a bad thing to do, but the obstetrician doesn't do it; he never suggests that drinking is the only way a child may end up with birth defects. Choice (A) is the best answer.

24. D. A university that licensed a low-cost online workshop on proper citation techniques for its incoming students found itself needing to commit significantly fewer valuable staff hours to handle incidents of unintentional plagiarism.

Look for an answer in which someone's preparation saves money by avoiding a more expensive problem that otherwise would have occurred. Choice (A) doesn't work because the homeowner isn't preventing a problem. Choice (B) doesn't work because it doesn't mention that measles were prevented via this measure. Choice (C) is an example of preparation, not prevention. Choice (D) does conform to the principle; the little amount of money spent on the workshop prevents the waste of valuable staff resources later. Choice (E) doesn't work quite as well. The problem wasn't headed off in the first place; it actually occurred. Choice (D) is the best answer.

25. C. A large number of studies have confirmed that the more anxious a patient is, the more protracted his or her recovery from a medical condition is.

The psychologist claims that patients do better when they like their doctors; the premises are three factors that result from liking your doctor. To support this argument, you need to find statements that link those factors with health improvements. Choice (A) doesn't help. It illustrates the danger of people not liking their doctors but doesn't prove the psychologist's point that patients who like their doctors are healthier. Choice (B) provides evidence of how the average doctor doesn't have a good relationship with patients, but it doesn't show the results of good doctor-patient rapport. Choice (C) does support the conclusion by linking one effect of liking a doctor (less anxiety) to a greater improvement in health. Choice (D) may suggest that improving doctor-patient communication is desirable, but it doesn't specifically link improved communication to improved patient health. Choice (E) presents a reason for poor doctor-patient communication but doesn't show how that affects patient health. Choice (C) is correct.

Section III: Reading Comprehension

1. D. Recumbent bicycles, which were banned from competition in the 1930s and thus neglected for much of the 20th century, offer many advantages over upright bicycles and come in many designs.

Think what this whole passage is about. You know that it describes a kind of bicycle called a recumbent that's designed differently from a regular bicycle, which the passage refers to as an “upright” bicycle. It mentions a bit about the history of recumbents and why they were banned from competition, describes the advantages of recumbents, spends a paragraph on different recumbent designs, and then concludes with a paragraph on how to improve recumbent hill-climbing. It's mostly informative, but the author definitely has an agenda — notice how she dwells on the advantages of recumbents.

Choice (A) is definitely wrong. The UCI's decision is only a minor part of the entire passage, not the main idea. Choice (B) isn't the main idea, either. Although the author does mention that recumbents have surged in popularity, which has generated new design interest, that's not the main idea of the whole passage; instead, it's kind of the main idea of the third paragraph. Choice (C) is true and is one of the author's points, but it's not her main idea, which has to apply to the entire passage. Choice (D) looks like the right answer. It sums up the gist of the entire passage pretty well and doesn't add anything that the passage doesn't say. Choice (E) is true and is also one of the author's points, but it doesn't sum up the main idea of the entire passage. Choice (D) is correct.

2. A. The seating position places the cyclist in a permanent aerodynamic tuck, with arms and legs contained within the rider's torso profile, thereby creating less wind resistance than an upright rider faces.

The passage makes it clear that aerodynamics are the reason for recumbent speed. Look for an answer that emphasizes the aerodynamic advantages of recumbents. Choice (A) looks good, but check the others. Choice (B) is wrong because the author explicitly says that most recumbents are made of steel, not of lighter materials. Choice (C) is a bit tricky because recumbents do maximize aerodynamics, and fairings and seat position do play a role in aerodynamics, but the main way that recumbents maximize aerodynamics is through an aerodynamic tuck. The author clearly states that fairings are optional, so you can't assume that fairings are responsible for the aerodynamic improvement. Choice (D) doesn't really work because you know that bicycle manufacturers neglected recumbents for most of the 20th century. Choice (E) doesn't work either; it's not specifically steering that makes recumbents faster, and one steering configuration is known to be faster than the other. Choice (A) is the right answer.

3. E. After a second-rate cyclist won a race and broke several records on a recumbent, the UCI decided that recumbents offered too much mechanical advantage and that, in order for races to reflect actual ability, all riders should use the same kind of bicycle.

The UCI (incidentally, Union Cycliste Internationale is a French name, though all the words resemble English; the English version is International Cycling Union) banned recumbents after a lousy cyclist won a race on one, proving that his bike was faster than the other bikes because he himself wouldn't have won on an upright bike. Choice (A) is wrong because the passage says nothing about other cyclists petitioning anyone. Choice (B) is totally wrong because the author never mentions anyone setting records on upright bikes. Choice (C) isn't in the passage. Choice (D) starts off promising with the mention of the second-rate cyclist but then veers into nonsense about press attention, so that's no good. That leaves Choice (E), which is the correct answer.

4. B. Recumbent bicycles are lighter than upright bicycles, which gives them an advantage at hill climbing.

Find the answer to this one by the process of elimination. If you prefer, mark the passage where you find answer choices that do appear. Choice (A) is mentioned; the author mentions comfort several times. Choice (B) looks wrong. The author says that recumbents are heavier than uprights, not lighter, so this answer certainly isn't one of the advantages she lists. Choice (C) is in the passage as an advantage; so are Choices (D) and (E). Choice (B) is correct.

5. A. to argue that, contrary to what they may expect, cyclists can climb hills well on recumbent bicycles, and to suggest ways to improve climbing performance

Before predicting the right answer, you may want to quickly skim this paragraph. Now, in your own words, why did the author write this paragraph? The last paragraph contains the author's discussion of climbing on recumbents. She mentions that some cyclists believe that recumbents are slow at climbing, claims that they aren't, and offers a suggestion or two of ways recumbent cyclists can improve their climbing prowess. Choice (A) matches up with that description quite well. Choice (B) is wrong, though it may be a bit confusing; the author does say that recumbent cyclists can climb hills well, but she isn't proving anything, just making a statement that she could back up with evidence if she chose to, though she doesn't. Choice (C) isn't exactly right. The author does discuss cadence, but that's not her primary purpose in writing the paragraph. Choice (D) doesn't work because, although the author does dispute the contention that recumbents can't climb, she doesn't offer anecdotal evidence; the claim that “most experienced recumbent riders” climb well isn't anecdotal evidence — you need some actual examples of fast riders for this answer to be right. Choice (E) is wrong; the author does recommend weight lifting, but that isn't the main point of the paragraph. Choice (A) is correct.

6. C. a short wheelbase bike with above-seat steering and a fairing

To answer this question, you have to use information from the passage. The third paragraph discusses the different designs of recumbent bikes: A short wheelbase is faster than a long wheelbase, and above-seat steering is faster than below-seat steering. If you go up to the second paragraph, you find mention of fairings, which can increase speed. So your fastest bike would be a short wheelbase bike with above-seat steering and a fairing. That's Choice (C).

7. E. A cyclist who wants to maximize speed and comfort and is not interested in racing should choose a recumbent over an upright bicycle.

You can't predict the answers to this one, so just read through the choices and see which one jibes with what the author seems to believe. Choice (A) doesn't really work because the author probably wouldn't make such a blanket statement. She's precise in her numbers and not likely to generalize like that, with no consideration for different riders. Choice (B) is wrong because, although the author mentions that recumbents have become more popular, nowhere does she mention absolute numbers of recumbent cyclists as compared to upright cyclists. For all you know, recumbent cyclists comprise a mere 1 percent of the cycling population. Because she hasn't mentioned this topic, you can't assume she'd agree. Choice (C) is wrong. A fairing would make a recumbent faster but by improving aerodynamics, not by reducing weight; if anything, the fairing would increase weight. Choice (D) is wrong. Even if the author conceded that a tucked position on an upright bike is as efficient as the normal position on a recumbent, she would never agree that it was as comfortable as sitting back in a recumbent seat. Choice (E) is correct; the author's points are that recumbents offer both speed and comfort, though it's true that you can't enter them in most bike races.

8. C. The diagnosis of PTSD is highly controversial.

Both authors overtly state that PTSD and the research and theories surrounding it are highly controversial. Choice (A) is mentioned only by the author of Passage B, and because you're only assessing the information found directly in the passage, you can't assume that the author of Passage B would necessarily agree. Same goes for Choice (E). Similarly, Choice (B) is mentioned by the author of Passage A but not by the author of Passage B. If you picked Choice (D), you may have missed the fact that the idea that trauma was limited to catastrophic events was referred to as a belief of the past, and more specifically, from the late ’70s and early ’80s.

Note: The two passages provide information about the diagnosis of PTSD as a medical disorder. Although both are relatively objective in purpose, Passage A seems more concerned with discussing the controversy surrounding whether PTSD is a legitimate diagnosis, and Passage B primarily describes the nature of PTSD symptoms.

9. A. PTSD: A Brief History of a Modern Controversy

Both passages begin with descriptions of how PTSD came to be recognized and then later cite how controversial the concept is, making Choice (A) the best of those listed. Though both note that some of the material on PTSD in the Diagnostic and Statistical Manual of Mental Disorders is indeed controversial, neither comes out and directly calls it false. Eliminate Choice (D) for the same reason — neither author implies that PTSD is a straight-up work of fiction. Choice (C) is pretty out of left field, so you can go ahead and knock that one out of contention. As for Choice (D), though the author of Passage B does note some common traits of PTSD sufferers, a “case study” implies a thorough analysis of a particular individual, and that is not present in either passage.

10. E. Sufferers of PTSD experience a lack of interest in day-to-day activities they used to find enjoyable.

Choice (A) is an argument made by the author of Passage A, when he says the concept of a traumatic stressor has broadened to such an extent that, today, the vast majority of American adults have been exposed to PTSD-qualifying events. Choice (B) is made by the author of Passage (B), when he states that “People with PTSD are consumed by concerns about personal safety.” Choice (C) is also described in the first paragraph of Passage B. Choice (D) is mentioned by the author of Passage A as one of two concerns of the past that have “again resurfaced in contemporary debates.…” As for Choice (E), remember that you need to cast aside anything you may personally know about PTSD and instead focus only on the material in the passage. Though Choice (E) may well be true, it isn't stated in either passage and therefore is the best choice of those available.

11. A. The author of Passage A suggests that PTSD may be an invalid diagnosis, while the author of Passage B indicates that PTSD is a valid disorder with recognizable symptoms.

In discussing the controversy over whether PTSD should be a medical diagnosis, Passage A uses diminishing terminology such as “merely” to describe the way that many symptoms have been combined to create the diagnosis of PTSD. This diminution implies that the diagnosis may not be a real medical condition but instead the invention of social and political activists. This recognition allows you to eliminate Choices (C) and (E). Passage B appears to accept that PTSD is a valid diagnosis and focuses on its symptoms. So you can cross out Choice (D). That leaves you with Choices (A) and (B). Passage B doesn't indicate that no other disorder exhibits PTSD's symptoms and in fact implies that two other disorders (ASD and AD) may have similar symptoms. Choice (B) must be wrong. Choice (A) is a better answer.

12. E. to categorize the primary symptom of PTSD as a problem with the manner in which its sufferers respond to their environment

Passage B begins with an assertion of the generally accepted cause of PTSD and links a stressor to its symptoms. It then goes into more detail about how the stressor contributes to PTSD symptoms and stresses the importance of the reaction of the PTSD sufferer to real or imagined dangers. You can eliminate answer choices that don't relate to why PTSD symptoms occur. Choice (B) regards an aspect of Passage A rather than Passage B. Choice (C) suggests that Passage B is about traumatic stress disorders in general instead of PTSD in particular. The passage doesn't compare PTSD suffers to trauma survivors who don't have PTSD, so Choice (A) isn't correct. Between Choices (D) and (E), Choice (E) is a better answer because the passage doesn't list causes or mention more than one general cause of PTSD symptoms. Choice (E) correctly summarizes Passage B's focus on the primary cause of PTSD symptoms, which is the way that sufferers react to stressful events.

13. C. the controversy over whether PTSD is a medical diagnosis is complicated by the motivations of the movement behind PTSD

Refer to the line reference and consider the context of the quoted words. The author of Passage A asserts that PTSD as a disorder is controversial and mentions the political and social construct to explain that some believe that PTSD as a disorder may have been created by psychiatrists who were against the Vietnam War and, therefore, PTSD isn't a medical disorder at all. You may be tempted by Choice (A) because it addresses the influence of antiwar psychiatrists, but you don't have enough information in the passage to assess the actual motives of the creators of the diagnosis. You don't know specifically from the passage that they were concerned with getting medical treatment for veterans. Choice (B) provides an opposing idea to the information in Passage A, and you know it's wrong because it suggests that something should be done. Descriptive passages are rarely calls for action. Choice (D) may sound good at first because it uses language straight from the passage, but Passage A doesn't talk about those who suffer from medical diseases from natural events. You know that Choice (E) is off base; Passage A doesn't focus on the controversy over PTSD's causes. The best answer is Choice (C). It notes that the context of the words in question relate to the controversy over whether PTSD is a real medical condition or a constructed one, and it indicates that its origination by a movement by antiwar psychiatrists calls into question its validity as a medical diagnosis.

14. C. Authority is a kind of power that does not explicitly depend on persuasion or coercion for its effect.

This passage is tough. A bit of strategy: You will of course have read the questions before tackling the reading, and you'll have noticed that this passage only has five questions. In this situation, you may be well served by postponing this reading until the end. It's not worth as many points as the others, and it's harder to understand. Often, it doesn't matter whether you work the passages in order, but readings like this can really bog you down.

For a clue as to its topic, read the last sentence of the first paragraph, a common place to put a thesis: it's about “the particular manifestation of power that we call authority.” This is an excerpt from a much longer piece on Alexander the Great, which is why Alexander appears in this passage, but Alexander isn't actually the focus of the passage; authority as a subcategory of power is.

Now, what's the central idea? The author is arguing that authority is a kind of power that relies for its effect on the speaker's ability to command and get people to obey him and doesn't depend on coercion or persuasion. Read through the answer choices and see whether you can find one that fits. Choice (A) doesn't work; the author has said that authority is a kind of power, which means they can't be the same thing. Choice (B) is backward — authority is a kind of power. You don't need to bother with the rest of the answer, which is also wrong. Choice (C) looks good. The author has said that “authority is a kind of power” and “for us to see authority in action, both coercion and persuasion must remain in the background,” which adds up to Choice (C). Choice (D) doesn't work; though the author mentions Alexander, he's really incidental to this discussion and even more so is his general, Antipater. Choice (E) is wrong, too. In the last paragraph, the author describes Alexander's exercise of authority but not of power. Choice (C) is correct.

15. B. explore aspects of the concept of authority and its uneasy association with coercive and discursive power

The second paragraph contains a discussion of authority and the relationship implied in any situation in which one person has authority over another. Choice (A) is wrong. Alexander doesn't appear in this paragraph. Choice (B) looks good — the author does mention “coercive power” and uses the word “uneasy” to describe relationships in which one person has authority over another. Choice (C) isn't right. The author may in fact suggest that coercive power is essential to authority, but that's not the point of the entire paragraph and is never really directly stated. Choice (D) is wrong. The author doesn't mention other scholars or their interpretations anywhere. Choice (E) is a statement that appears in the third paragraph, not the second, and it isn't the point there, either. Choice (B) is the best answer.

16. C. hidden

The sentence in question reads: “But for us to see authority in action, both coercion and persuasion must remain in the background, occulted.” If something is in the background, you're not supposed to see it, or at least not focus on it. Choice (C), “hidden,” would make sense in this context. The other choices don't. The only other choice that may make some sense is Choice (B), but “hidden” makes much more sense than “understated.” Incidentally, although the word occult can refer to magic, occulted in this context has nothing to do with magical topics, so Choice (D) is way out there. Choice (C) is correct.

17. B. Power is a generic term for the ability to make someone do something; authority is a particular form of power, the ability to command without explicit persuasion or coercion.

The definition of power appears in the first paragraph, and the definition of authority appears in the second. Note the author's use of the word “generic” to describe “power.” Skimming the answers, Choices (B) and (E) look like possibilities. Choice (B) looks right and Choice (E) looks wrong; the author says that authority is a kind of power and that it shouldn't depend on outright coercion or persuasion. Choice (A) is wrong because both authority and power can involve persuasion and coercion. Choice (C) doesn't work because nowhere does the author suggest that authority goes with politics and power goes with the military. Choice (D) is likewise wrong. Choice (B) is the best answer.

18. A. The speaker's authority becomes weaker, whereupon he can resort to persuasion or force, or reassert his authority by invoking his privileged position.

Here's what the author says: “If the listener demands a reason for a command or asks about the consequences of disobedience, the speaker's authority falters. At this point, authority may give way to persuasion or naked force, or it may be reasserted by invoking the privileged, authoritative position of the speaker.” Choice (A) looks like it matches up pretty well with that. Choice (B) is wrong because the speaker doesn't necessarily lose power but may lose authority. In addition, you should automatically be suspicious of extreme wording such as “loses all power.” Choice (C) is just plain wrong. Choice (D) isn't right either, because the speaker could also use persuasion or reinvoke his authority. Choice (E) definitely isn't right. Choice (A) is the best answer.

19. E. The Panama Canal, built as a result of military strategists worried about transporting Navy ships from one ocean to another, not only expedited military transport but also made moving cargo between the oceans easier, opening a wealth of economic opportunities.

This passage is a short history lesson on the Panama Canal — the events that preceded its construction and the worries they caused, and then the effects on the world after it was operational. Choice (A) isn't the main point, though it is a factoid you can figure out from the passage. Choice (B) isn't the main point, either; the passage isn't only about the U.S. Navy's use of the canal during World War I. Choice (C) sums up the first half of the passage but doesn't do justice to the whole thing. Choice (D) is a conclusion that the author doesn't state. Choice (E) is the best answer; it sums up the passage nicely.

20. D. Without the canal, the only way to transport boats from the Atlantic to the Pacific was to sail them around the entire continent of South America, which took more than two months.

Why did the United States decide it needed the Panama Canal? The only way to get ships from the Atlantic to the Pacific or vice versa was to sail around the entire continent of South America. The narrow Isthmus of Panama (an isthmus is a little strip of land with ocean on either side of it) was an obvious place to construct a passage to shorten the trip. Choice (A) isn't right; the Navy had no desire to build two complete fleets. Choice (B) is true, but the passage gives the impression that the impetus behind the construction was military, not commercial. Choice (C) is misleading because it's the event that starts off the passage, but you're looking for a logistical problem, and having a ship blown up doesn't fit that category. Choice (D) looks right. The problem was indeed that it took too long to move ships from one ocean to another. Choice (E) is similar to Choice (C) in that it mentions the event that begins the passage and led to worry by military strategists, but the single instance of lacking a ship wasn't the logistical problem that led to the digging of a major canal. Choice (D) is correct.

21. B. The U.S. Navy was unable to use all its ships in the same conflicts because it took far too long to sail ships from the Atlantic to the Pacific and vice versa.

Here's what the passage says: “The impossibility of sailing north of North America, and the distance of the voyage around South America, effectively divided the U.S. Naval forces into two fleets. Such a division was costly, inefficient, and ultimately less powerful than a fleet that could unify on demand.” What that phrase means, then, is that without a shortcut between oceans, the Navy essentially had to keep one fleet in the Pacific and one fleet in the Atlantic; ships couldn't necessarily sail to help one another because the trip took so long. Choice (A) is wrong. Naval commanders certainly wished they could mobilize their entire fleet in the same place at the same time. Choice (B) looks correct. Choice (C) doesn't work; the USS Oregon did in fact make it to Cuba in time to play a pivotal role. The problem was that the Navy couldn't count on this happening in the future, but the trip wasn't impossible. Choice (D) isn't right; it describes one incident that exemplified the two-fleet problem but doesn't sum up the phrase in question. Choice (E) doesn't work either because the author never mentions air power. Choice (B) is right.

22. C. Spain could have won the fight for Cuba.

After the USS Maine blew up, the Navy wanted to send its brand spanking new battleship to Cuba, but the author points out that the Navy had to wait 67 days for it to get there, with naval officers undoubtedly biting their nails the whole time. Why were they nervous? Spain and the United States were fighting over Cuba. The author doesn't say exactly what was going on, but you can assume that if the USS Oregon hadn't arrived on time, Spain would have taken Cuba. (In fact, Cuba had started a war for independence from Spain, and the United States sent the Maine to assist the Cubans. After the war, Cuba was independent. But you don't need to know that to answer the question.) Skim the answers; Choice (C) looks exactly right. Choice (A) may be true, but the author doesn't imply it. Choice (B) is wrong because, although the loss of more ships would have been undesirable, that wasn't the nation's primary concern. Choice (D) is irrelevant. You don't have any reason to assume San Francisco needed defense. Choice (E) may also work, but that's hardly what the passage suggests the Navy was worried about (surely the Navy would have found another opportunity to use the Oregon). Choice (C) is the best answer.

23. A. It has transformed Panama into a major player in world trade.

Read through the passage and mark off the answer choices that you find. Choices (B), (C), (D), and (E) are all in there, mostly in the last paragraph. Conspicuously absent from the author's discussion of the canal's impact is its effect on its home nation, Panama. Choice (A) is the answer.

24. E. describe the drawbacks of sending ships around South America and the reasoning that led to the decision to build the Panama Canal

In the second paragraph, the author discusses the concerns and desires the Navy had that led to the construction of the Panama Canal. Choice (A) is wrong because, even though the thinking of military strategists on the subject of the Spanish-American War does appear, that's just a piece of evidence that backs up the main point of the paragraph. Choice (B) is misleading because the paragraph starts off with the Oregon’s contribution, but the battleship never appears again in that paragraph, so it's not the primary purpose. Choice (C) is wrong. The action in the second paragraph occurs before the canal was built, so it can't mention the costs and benefits of traveling through it. Choice (D) is wrong. There weren't any effective alternatives to the canal. Choice (E) is correct. The military couldn't help but notice that South America was awfully big and time-consuming to sail around and reasoned that a canal through Panama would speed things up nicely.

25. B. to describe the history behind the construction of the Panama Canal and the canal's importance to world trade and the U.S. military

This is an analytical history passage, describing events and interpreting the forces behind the decisions of the time. Choice (A) isn't right. The author doesn't really get into the business of justifying U.S. actions in building the canal. Choice (B) looks good. Choice (C) is wrong because the passage never mentions imperialism. Choice (D) is wrong because the author doesn't get into discussing the construction of the canal. The passage skips from military strategists worrying in the late 1800s to the use of the canal in World War I, and you never find out about the details of construction. Choice (E) is wrong, too, because explosion of the Maine is just incidental to the story, not the author's primary concern. Notice that you don't see anything about the Maine after the first paragraph. Choice (B) is the best answer.

Section IV: Logical Reasoning

1. B. People who don't have children attending public schools cannot benefit in any way from the existence of public schools.

The curmudgeon assumes that the only way a person can benefit from public schools is by having children who attend them. Choice (A) is wrong. Although Choice (A) presents a possible benefit from public education, it goes too far to be the curmudgeon's basic assumption. Remember to stay close to the text — an assumption is the most basic piece of evidence that bridges the stated evidence to the conclusion. Choice (A) goes a step beyond the assumption and makes another assumption — that education is necessary to vote intelligently — but the curmudgeon never mentions voting or democracy in her argument. Choice (B) looks like the right answer; she does believe a childless person can't benefit from public schools, and that belief links the premise of the curmudgeon's argument to her conclusion that public schools don't benefit her. Choice (C) is probably something the curmudgeon would agree with, but it's not the assumption that ties together the premise to the conclusion. She mentions nothing about other people getting undeserved benefits. Choice (D) is totally wrong because it has nothing to do with the argument. Choice (E) is a point that someone arguing with the curmudgeon may bring up, but it's not the curmudgeon's assumption. Choice (B) is the only correct answer.

2. D. misinterprets the results of the research by failing to note that the sleep technique suggested by scientists requires the sleeper to remove the socks before trying to sleep

Focus on the actual conclusion: Insomniacs should try wearing socks to bed. Choice (B) isn't the flaw; the magazine suggests only that insomniacs try wearing socks, not that sock-wearing provides an absolute cure for sleeplessness. Choice (A) is wrong because the magazine justifies the technique with reference to scientific research. Choice (C) isn't a flaw in reasoning; on the LSAT, a reference to scientific research is usually sufficient documentation for a claim. Choice (E) is wrong because the magazine doesn't criticize insomniacs. However, the conclusion leaves off the crucial step of removing the socks before bed to allow the feet to cool off — that's its mistake. The magazine misinterprets the study's results and should have added that, for the technique to work properly, the feet must first be warmed and then be allowed to cool off, such as by removing socks. Choice (D) looks like the best answer.

3. D. is based on a generalization that is not supported with evidence

This argument is a perfect example of an overly broad prescriptive statement. “Everyone knows” assumes general agreement without evidence to support the claims. Choice (A) is wrong because the speaker offers no evidence to specifically discount. Choice (B), if true, may be something to bring up in an opposing argument, but this kind of real-world rebuttal isn't a flaw in the internal logic of the author's argument (and in any case, “many” would be too weak to do much damage). Choice (C) is wrong; the argument doesn't talk about women in the workplace. Choice (D) looks right. The author makes a generalization that all mothers should stay home and offers only another generalization (that everyone knows mothers love their babies) and a restatement of the conclusion (mothers should raise their children) to support it. Choice (E) reaches beyond the score of the argument. The author doesn't specify potential caregivers other than mothers. Choice (D) is the best answer.

4. C. Some chimpanzees are capable of understanding more than 200 words.

Did you take a good look at the question before you started reading the statements? You're looking for something that must be true. Underline that word so that you know what the right answer will look like. Now that you know what you're looking for, do some quick thinking before going on to the answer choices. What must be true? You may think something like, if dogs can understand as many words as chimps, dolphins, and parrots, and dogs can understand 200 words, then chimps, dolphins, and parrots can understand at least 200 words. Choice (A) may be true, but it doesn't have to be true based on the information contained in the argument. You can't get to Choice (B) from the argument, which doesn't address training techniques. Choice (C) looks like a logical conclusion and a good answer. If many dogs can understand more than 200 words, and intelligent animals (chimpanzees in this case) are capable of understanding no fewer words than dogs, you can indeed deduce that at least some chimpanzees are capable of understanding more than 200 words. Choice (D) misses the point. The researchers may have been in Germany, but the animals could be from anywhere. Choice (E) isn't a conclusion you can make because the author never mentions training methods, only results. Choice (C) is right.

5. C. My neighbor wants me to give her plant a gallon of water every day. I will not have to water it as often if I give it more water each time, so I am going to give it 2 gallons of water every other day.

Yikes — burnt pie. The reasoning in the argument goes like this: 40 minutes at 250 = 20 minutes at 500. You want to find an answer that results in more or less an identical flaw: the author mistakenly assuming that a time-sensitive process can be halved in duration simply by doing it twice as intensively. Choice (A) goes like this: one aspirin every four hours; I'll do two aspirin every four hours. That's not a match. Choice (B) states that 9 pounds gets 1 cup; 13 pounds gets 1.5 cups. That reasoning seems sensible and not flawed. You're looking for a flawed argument, so Choice (B) won't work. This answer may look attractive to you because it manipulates the proportions in the directions in a similar way to that in the argument, but remember that the argument messes up that manipulation; the cat-feeder here presents the same flawed calculation as the original. Choice (C) reasons that 1 gallon every day = 2 gallons every other day. That's very close to the reasoning in the argument, which doubled the temperature and halved the time. Choice (D) is like this: bleach for 20 minutes; I'll do bleach for 40 minutes. The quantity of bleach isn't affected, so that argument isn't flawed in the same way (but that doesn't mean you should apply that reasoning to your own hair!). Choice (E) figures that twice as fast is worth twice the cost, which isn't necessarily flawed reasoning. Choice (C) is the best answer.

6. B. the goodwill and publicity generated by charitable giving can increase a business's sales significantly

This author assumes that there's no possible connection between charitable giving and profit-making, so you're looking for an answer choice that suggests such a link that's been overlooked in reaching the extreme conclusion on hand here. Choice (A) is attractive but wrong because, for all you know, “some” deductions could be quite minor. Choice (B) is correct because it suggests a direct link between charity and profits and thus calls into question the businessman's assumption that charitable giving makes no sense. Choice (C) is irrelevant. A flaw in the businessman's reasoning will be an unjustified conclusion based on the evidence he's given; statements that quibble with that evidence aren't flaws in his logic. Choice (D) may be nice, but the argument isn't about investors and mutual funds. Choice (E) also conveys an altruistic sentiment, but it's not clear that the charitable giving at these “some businesses” is driving their profits or detracting from it. Choice (B) is the best answer.

7. A. Though average weights have increased, the weight gain has occurred exclusively among people who were already overweight; thin people have not gained weight, despite the fact that they, too, are consuming more fast food than was the case two decades ago.

The nutritionist blames fast food for the obesity epidemic. Any evidence disproving a connection between fast food and obesity would weaken the argument. Choice (A) looks like it may do that; if everyone is eating more fast food than before but only certain people are getting fatter, then eating fast food alone can't be the cause. Choice (D) provides another possible reason for childhood obesity but doesn't speak to the rise in obesity in the population overall. Choice (C) actually reinforces the argument that fast food is the culprit. Neither Choice (B) nor Choice (E) addresses the fast food–obesity connection, so Choice (A) must be the answer.

8. B. All flies are insects. All insects are arthropods. All arthropods are invertebrates. Therefore, all flies are both arthropods and invertebrates.

This argument lends itself to being written as an equation: H = P; P = M; M = V; H = M and V. Try writing the answers as equations to see whether one comes out the same. Choice (A) goes like this: D = C; C = M; C = V; D = M and V. That's so close but not quite there; canines appear twice at the beginning of a sentence, which ruins this one. Choice (B) states that F = I; I = A; A = I; F = A and I. That looks identical to the argument, so it's a good answer. Choice (C) reasons that W = C; C = M; M have mammary glands; W have mammary glands. Nope. Choice (D) goes like this: S = M; M have shells; O = bivalves; bivalves have shells; creatures with shells = bivalves and mollusks. Totally wrong. Choice (E) indicates that G = P; C = P; P = V; G and C = V. That's not it. Choice (B) is correct.

9. A. It is permissible for a soldier to refuse to obey an order to torture and abuse prisoners of war because that would violate national and international law.

Look for an answer in which a soldier refuses to carry out an order he knows is against the law. Choice (A) looks right. Torture violates the law, so a soldier can legitimately refuse to carry out torture orders. Choice (B) isn't right. Soldiers aren't allowed to refuse orders that are simply against their own consciences. Choice (C) is wrong. A soldier's belief that the military shouldn't be involved in a particular conflict isn't the same as violating an actual law. Choice (D) is wrong for the same reason. Choice (E) is also wrong. For the answer to be right, it has to say that the action is against the law, not simply against an individual's conscience or moral beliefs. Choice (A) is the right answer.

10. B. It is impossible to pass laws prohibiting obscenity without violating the free speech rights of those producing the material.

Your job for questions that ask for the main point is to pick out the conclusion in the argument. The commentary's conclusion is that defining obscenity without violating free speech guarantees is impossible. That's actually stated in the first sentence. On the LSAT there's no reason why an argument can't begin with the conclusion (its “main point”) and then provide support for it. Choice (A) isn't the theorist's point; she doesn't mention the different roles of federal and state governments. Choice (B) works because it states exactly the end point of the theorist's reasoning, her main point. Choice (C) is wrong. The theorist doesn't suggest that courts and legislatures ignore all speech, just obscenity. Choice (D) is the final premise of the theorist's evidence, so it's not the main point. Remember, just because a statement occurs at the end of an LSAT argument doesn't mean it's automatically the conclusion. Choice (E) doesn't work because it's simply another piece of evidence; the main point builds on this premise to reach its conclusion. Choice (B) is correct.

11. D. Most consumers want to feel sure that the employees of businesses they patronize are not abusing serious drugs on the job.

Make sure you know what you're looking for: Look for the one answer that doesn't hurt the argument. It doesn't necessarily have to strengthen the conclusion, but it can't weaken it. What's the conclusion? That the best method to avoid drug-using employees from making mistakes on the job is to administer random urine tests. And this is based on the fact that these tests can detect certain drugs. Just go down the list and eliminate any answer choice that does weaken the link from this premise to conclusion. The last choice standing is right! Choice (A) weakens the argument that urine tests are the best method by suggesting hair tests are better. Choice (B) weakens the argument by suggesting that urine tests can't spot drugs taken more than a couple of days ago. Choice (C) weakens the argument by suggesting that random drug tests are mostly useless and thus unlikely to be the best method (the part about their cost, by the way, is irrelevant to this argument). Choice (D) doesn't weaken the argument. What consumers want is irrelevant to the success of urine tests in best detecting employee drug use. Choice (E) weakens the argument. If it's easy to cheat on urine tests, they can't be very effective. Choice (D) is the best answer.

12. E. A seller is obligated to observe the terms of the contract of sale, even if doing so works to the seller's own disadvantage.

Albert would argue that he was acting as the catalog told him he could act and therefore isn't taking unfair advantage of the seller. In other words, he is abiding by the terms of the agreement, as is the seller, so he isn't taking advantage of the company. Choice (A) states only that buyers and sellers enter unwritten contracts during transactions, but that doesn't justify Albert's actions, at least not completely; look for something better. Choice (B) is wrong. Albert takes advantage of the seller's goodwill by all evidence with the seller's consent. Choice (C) doesn't work because it doesn't bolster Albert's rationale to his wife; if anything, it undercuts it by calling it an abuse. Choice (D) doesn't justify Albert's position, though it's a reasonable warning to merchants. Choice (E) is the best answer. When Albert bought the shoes, the sale contract stated that he could have them replaced at any time, so he's only observing the contract's terms, as in fact the seller is. Choice (E) is the best answer.

13. B. A school district in another state reduced class sizes from 30 to 25 students; within a year, average test scores had increased by 25 percent.

Heloise doesn't believe that reducing class size makes any difference. See whether you can find an answer providing evidence that it does. Choice (A) actually helps Heloise make her point. If the Japanese thrive on very large classes, size may not be a problem. Choice (B) provides statistical evidence that does undermine Heloise's position. If reducing class sizes made a sizable difference in a short time, maybe it's valuable. Choice (C) doesn't really undermine her point. Teachers may like small classes better, but that doesn't mean they're better for students. Choice (D) would, if anything, help her point, though it's really irrelevant to the issue of student performance. Choice (E) could undermine her point if it went further, but the mere fact that private schools limit class size tells you nothing about the performance of the students in the smaller classes (and what you think you may know about private school students’ performance in the real world wouldn't ever matter on the LSAT). Choice (B) is the best answer.

14. E. Despite the large number of lost bags, airlines only lose the bags of 4 out of every 1,000 passengers, so the odds of arriving without checked luggage are very low.

That's a lot of lost luggage, but maybe the problem isn't as serious as the numbers make it seem. Choice (A) doesn't weaken the argument; if anything, it strengthens it. If airlines warn passengers to pack carry-ons, they must expect luggage to get lost. Choice (B) also strengthens the argument because it suggests that lost checked luggage can be expensive and inconvenient. Choice (C) doesn't weaken the argument for the same reason. Choice (D) doesn't really weaken the argument; although some travelers may not mind having to go shopping, many do, and luggage can contain more than just clothes. Choice (E) does weaken the argument that travelers shouldn't check their bags. If the odds of losing luggage are that low, then checking bags is fairly safe. Choice (E) is correct.

15. E. LASIK is usually successful at improving a patient's vision, but it does not always result in perfect uncorrected eyesight and can create new problems for the patient.

The information is about the benefits and risks of LASIK. Choice (A), while attractive, isn't a conclusion you can safely draw from the facts; though the passage says night vision can be impaired, the information says nothing about it being so bad as to prevent driving. Choice (B) doesn't work; the passage doesn't address the differences in doctors. In fact, the passage says the risks apply to all doctors. Choice (C) isn't supported by the passage. The passage tells you nothing about reading glasses, and in fact suggests that most patients need no glasses afterward. Choice (D) isn't in the passage either; the paragraph doesn't address aging. Choice (E) makes the most sense. According to the passage, LASIK has many benefits but some real risks. That actually makes a pretty good conclusion, summing up the information rather nicely. Choice (E) is the right answer.

16. A. Turning on the lights inside a building makes it easier for birds to see the glass windows and avoid flying into them.

You can't predict this answer, so dive in and find the one that outright contradicts (“must be false”) the information in the passage. Choice (A) looks like the right answer; the passage tells you that birds can't see glass when lights are on inside. Choice (B) is probably true if birds are running into buildings in record numbers. Choice (C) could be true based on the information in the passage. Nothing in the passage indicates whether Choice (D) could be true or false. Choice (E)'s statement is possible; the passage says the bird-building problem occurs during migration. Choice (A) is the only necessarily false statement and is therefore correct.

17. D. Outsourcing to foreign countries is not nearly as important a cause of layoffs as many Americans mistakenly believe.

The statements suggest that outsourcing to foreign countries isn't nearly the problem many Americans think it is. Choice (A) is wrong. The author doesn't think anyone should blame the government for letting jobs go overseas; instead, she suggests that foreign outsourcing is a fairly small factor in unemployment. Choice (B) sounds like it may work, but continue to check the remaining answers. Choice (C) is wrong. Although this fact about outsourcing may be true, it's not substantiated by information in the passage. Choice (D) looks right; that's exactly what the statements suggest, and it's more specific than Choice (B) because it mentions foreign outsourcing specifically. According to the passage, about three-quarters of 9 percent of laid-off workers lost their jobs to local outsourcing, so outsourcing is a possible, albeit remote, concern. Choice (E) may be a true statement, but nothing in the argument would require it to be true. Choice (D) is the best answer.

18. D. Children who work cannot go to school, and without an education, they cannot get high-paying jobs as adults, so their own children are forced to work instead of attending school.

First, be sure of what the question is asking. You're looking for an explanation of a phenomenon. If child labor is both a problem of poverty and a cause of poverty, there must be some sort of vicious cycle going on, probably caused by the fact that people are working as children and passing their disadvantages on to their own children. Choice (A) is only a partial answer, and introducing the social safety net is irrelevant. Choice (B) doesn't make sense. It sounds as though parents have a choice of whether to send their kids to school or work. Look for something better. Choice (C) may be true, but it doesn't totally explain how child labor and poverty feed on each other without filling in several more steps from health to poverty. You want a tighter explanation. Choice (D) explains the phenomenon best; that's a real circular problem. When uneducated children are unable to earn much, the situation forces their own children to be uneducated child laborers. Choice (E) doesn't really explain the cycle of poverty. Choice (D) is the best answer.

19. B. Reducing fossil-fuel emissions will slow global warming.

The environmentalists like the emissions-reduction laws because they think such laws will slow global warming; clearly they're assuming there's a connection between the two. Although this connection may seem obvious, the passage never explicitly makes the link, so it's an unstated assumption. Choice (A) is wrong because the author says nothing about coastal states being environmentally conscious. Choice (B) looks exactly right. Choice (C) doesn't come up. Choice (D) is interesting, and the author does touch on environmental priorities, but it's not the assumption that leads from premises to conclusion. Choice (E) isn't something you can conclude from the information given. Choice (B) is correct.

20. D. arguing that an alternate solution would have better dealt with the problem Philip addresses

Philip thinks hybrids are the greatest thing since sliced bread, but Ryan's not so impressed. He thinks older technology was a better solution than heavy, complicated hybrids and their weird batteries. Look for an answer choice that states Ryan's response. Choice (A) is wrong because Ryan, if anything, also seeks a solution to the problem of inefficient cars. Choice (B) is wrong because Ryan doesn't offer an analogy; he sticks with the issue of cars. Choice (C) is wrong because Ryan doesn't question Philip's facts about hybrids; he just offers different facts about older cars that he thinks are more pertinent. Choice (D) gets right to Ryan's approach of offering a better solution. Choice (E) is wrong because Ryan's response doesn't contain a counterexample to disprove Philip's reasoning. Choice (D) is the best answer.

21. D. families may not consist of only females

Pro-family brochures distributed only to women? Though many women may not want to get married, that's not the main flaw in the reasoning, so Choice (A) is wrong. Choice (B) may be true but not because of flawed reasoning. Choice (C) may also be true, but the legislator isn't thinking only of single mothers. Choice (D) does name a flaw; any efforts to improve the family need to recognize that women aren't the only people in them. Choice (E) may well be true, but it's not the main reason the thinking is flawed. Choice (D) is the best answer here.

22. B. The traffic between Tokyo and Yokohama was so bad that Stuart could not go faster than 25 kph.

The best explanation for this otherwise surprising result would be something that suggests that Stuart's car couldn't get close to the allowed maximum speed of 60 kph. Choice (A) explains how Steve kept moving but doesn't account for Stuart's slow performance. Choice (B) could well offer an explanation; if the traffic was terrible, Stuart couldn't go any faster than Steve. Choice (C) doesn't help explain matters. If Stuart was driving 60 kph, he should have arrived sooner, unless he couldn't achieve that speed often, but the answer doesn't tell you how often he could go 60kph. Choice (D) doesn't help either because, all things being equal, Stuart should have arrived much earlier. Choice (E) had to be frustrating for the two drivers but still doesn't answer the question. Stuart stopped more, but he should have been able to drive faster when he was moving. Choice (B) makes the most sense.

23. C. Uncontrolled anger is a cause of road rage.

This judge must be assuming that there's some connection between controlling anger and eliminating road rage. It seems pretty obvious but isn't explicitly stated, so it counts as an assumption. Choice (A) is too extreme. The judge says that yoga can help control anger but doesn't suggest that yoga practitioners never lose control. Choice (B) isn't the reason for the sentence of yoga. The judge is trying to improve the man's behavior, not just impose a more serious sentence. Choice (C) looks like a good answer because the judge clearly assumes that yoga, which apparently helps control anger, is an apt punishment for a road rage defendant. Choice (D) may be tempting, but yoga isn't offered as a fitting payback for road ragers; it's offered as an attempted cure. Choice (E) is wrong. The judge doesn't give the impression that road rage is justified in this case. Choice (C) is the best answer.

24. C. Despite the popularity of online dating services, participants should realize that such services come with many potential pitfalls.

To predict the conclusion, think about where the evidence is leading you. Try to compose a sentence that would follow the last sentence in the argument, beginning with the word “therefore.” The argument's natural conclusion is that online dating must be approached with caution. Choice (A) isn't the conclusion but a piece of evidence the author mentions. Choice (B) is too specific to be the conclusion, though the author does suggest that sexual predation may be a danger. Choice (B) expresses a detail instead of the general conclusion of the entire prompt. Choice (C) looks like a good answer; it summarizes the author's argument pretty well. Choice (D) isn't the author's point; he's not recommending that online dating services take any specific actions. Choice (E) is a prediction about the future of online dating that can't be inferred from anything in this passage, so it isn't its main conclusion. Choice (C) is right.

25. D. An artwork's meaning is dependent on contingent features of the artist's psyche and society rather than on any inherent quality of the work itself.

To best approach this question, notice that the postmodern reaction is against formalism, which the art critic defines as an emphasis on an artwork's form rather than its content or subject matter. So eliminate principles that justify form over content. Choices (A) and (C) emphasize the form of an artwork rather than its content, so those principles undermine rather than justify the reaction against formalism. Neither the formalist view nor the postmodern reaction mentions how personal taste relates to art evaluation, so Choice (B) is irrelevant. The remaining answers, Choices (D) and (E), appear to contradict each other. Choice (E) suggests that art loses its inherent (inner) value when people attempt to explain it or give it external meaning, and Choice (D) asserts that art's message comes from the artist's social and psychological experiences, which suggests that the artist intends a piece of art to mean something beyond its form. The principle that an artist projects meaning through artwork is more consistent with the postmodernist view that art should convey an interest in worldly affairs. Therefore, Choice (D) most justifies the postmodern reaction against formalism.

Example Essay

The dilemma these two sisters face involves determining whether a trip to Ireland or a Dominican Republic visit would better accomplish their vacation goals, which are to spend as many days as possible on horseback and improve their riding skills under the instruction of a qualified teacher. The explanation of the sisters’ travel options plainly states that should they select the Ireland adventure, they would have only one week to spend honing their skills with the assistance of qualified instructors. The second option of vacationing in the Dominican Republic would provide them with twice as much time on horseback, but they would not be exposed to the professional instruction they desire. Because the trip to the Dominican Republic would afford the sisters a longer vacation and therefore more days riding horses for essentially the same price as the one-week trip to Ireland, it is a better option. Though the alternative would provide them with arguably better equitation instruction, the sisters would gain that knowledge at the expense of spending twice the hours on horseback.

The Dominican Republic trip would allow the sisters the opportunity to fulfill their wish to maximize their riding time. There they would enjoy a full seven additional days on horseback with the freedom to choose the number of hours they spend riding and the terrain they cover. The experienced guide can surely provide the women with informal tips and tidbits to help them improve their riding ability, and the extended time will give the sisters ample time to hone their equestrian skills before the pre-med sister becomes involved in her studies.

Although the sisters would likely gain more specific equitation expertise from the visit to Ireland, the one-week time frame severely limits their ability to fully benefit from that instruction. The women will likely require a day to acclimate to the time change, which leaves only six riding days for them to enjoy. Additionally, because qualified riding instructors are not exclusive to Ireland, the sisters may find equally qualified riding instructors on their home soil to help them develop their skills upon returning stateside from the Dominican Republic. Finding another window of time where both sisters could sneak away for two weeks before the one engages in rigorous coursework may prove more difficult than carving out several hours to take riding lessons at home.

Thus, because neither option both maximizes riding time and includes training from a qualified instructor, I would advise the sisters to take the longer of the two trips, which is to travel to the Dominican Republic. There, they can spend as much time as possible on horseback, improving their skills as a result. They can always brush up on their technique with an instructor back home at a later time.

Answer Key for Practice Exam 1

Section I: Analytical Reasoning

1. C

2. C

3. C

4. B

5. C

6. E

7. D

8. E

9. D

10. A

11. E

12. D

13. B

14. B

15. A

16. A

17. C

18. B

19. D

20. D

21. A

22. C

23. E

24. D

25. C

26. E

Section II: Logical Reasoning

1. D

2. C

3. D

4. C

5. A

6. C

7. D

8. A

9. E

10. B

11. D

12. C

13. D

14. A

15. E

16. C

17. B

18. B

19. A

20. E

21. D

22. E

23. A

24. D

25. C

Section III: Reading Comprehension

1. D

2. A

3. E

4. B

5. A

6. C

7. E

8. C

9. A

10. E

11. A

12. E

13. C

14. C

15. B

16. C

17. B

18. A

19. E

20. D

21. B

22. C

23. A

24. E

25. B

Section IV: Logical Reasoning

1. B

2. D

3. D

4. C

5. C

6. B

7. A

8. B

9. A

10. B

11. D

12. E

13. B

14. E

15. E

16. A

17. D

18. D

19. B

20. D

21. D

22. B

23. C

24. C

25. D

Computing Your Score

Add up the total number of correct answers from each section; if you got everything right, you should have a raw score of 101.

Section I __

Section II __

Section III __

Section IV __

Now compare your raw score to the following chart. That would be your LSAT score if this were a real LSAT. It's not, so whatever you get on this test should just give you an idea of your abilities.

table

remember.eps The real LSAT has an extra experimental multiple-choice section thrown in the mix, but you can't identify it. It looks just like the rest, and it doesn't count toward your final score.

tip.eps Wrong answers don't subtract from your raw score on the LSAT. So, if you run out of time, make sure you bubble in an answer for each question. You may get some right just by sheer luck!